Last visit was: 26 Apr 2024, 20:21 It is currently 26 Apr 2024, 20:21

Close
GMAT Club Daily Prep
Thank you for using the timer - this advanced tool can estimate your performance and suggest more practice questions. We have subscribed you to Daily Prep Questions via email.

Customized
for You

we will pick new questions that match your level based on your Timer History

Track
Your Progress

every week, we’ll send you an estimated GMAT score based on your performance

Practice
Pays

we will pick new questions that match your level based on your Timer History
Not interested in getting valuable practice questions and articles delivered to your email? No problem, unsubscribe here.
Close
Request Expert Reply
Confirm Cancel
SORT BY:
Kudos
Tags:
Difficulty: 555-605 Levelx   Assumptionx               
Show Tags
Hide Tags
Manhattan Prep Instructor
Joined: 30 Apr 2021
Posts: 521
Own Kudos [?]: 486 [1]
Given Kudos: 37
GMAT 1: 760 Q49 V47
Send PM
GMAT Club Verbal Expert
Joined: 13 Aug 2009
Status: GMAT/GRE/LSAT tutors
Posts: 6923
Own Kudos [?]: 63674 [1]
Given Kudos: 1774
Location: United States (CO)
GMAT 1: 780 Q51 V46
GMAT 2: 800 Q51 V51
GRE 1: Q170 V170

GRE 2: Q170 V170
Send PM
Tutor
Joined: 15 Nov 2013
Posts: 97
Own Kudos [?]: 925 [1]
Given Kudos: 3
GMAT 1: 800 Q51 V51
GRE 1: Q170 V170

GRE 2: Q170 V170
Send PM
Manager
Manager
Joined: 28 Jun 2015
Posts: 59
Own Kudos [?]: 51 [0]
Given Kudos: 163
Location: Australia
Send PM
Re: Editorial: Our city's public transportation agency is facing a budget [#permalink]
Could an expert please weigh in?

Q1.
A. The transportation employees' union should not accept cuts in retirement benefits if doing so would not be in the employees' best interest.
E. The transportation employees' union will not accept cuts in retirement benefits if doing so will not allow more transportation employees to keep their jobs.

In A and E, does "doing so" stand for ACCEPTING or NOT ACCEPTING?

Q2.
Could someone please try negating A and E? Negating them seems to hurt the argument.

Thanks and regards

Bunuel wrote:
Editorial: Our city's public transportation agency is facing a budget shortfall. The fastest growing part of the budget has been employee retirement benefits, which are exceptionally generous. Unless the budget shortfall is resolved, transportation service will be cut, and many transportation employees will lose their jobs. Thus, it would be in the employees' best interest for their union to accept cuts in retirement benefits.

Which of the following is an assumption the editorial's argument requires?

A. The transportation employees' union should not accept cuts in retirement benefits if doing so would not be in the employees' best interest.
B. The only feasible way for the agency to resolve the budget shortfall would involve cutting transportation service and eliminating jobs.
C. Other things being equal, it is in the transportation employees' interest to have exceptionally generous retirement benefits.
D. Cutting the retirement benefits would help resolve the agency's budget shortfall.
E. The transportation employees' union will not accept cuts in retirement benefits if doing so will not allow more transportation employees to keep their jobs.

CR11741.01
OG2020 NEW QUESTION
Intern
Intern
Joined: 27 Aug 2017
Posts: 4
Own Kudos [?]: 0 [0]
Given Kudos: 29
Send PM
Re: Editorial: Our city's public transportation agency is facing a budget [#permalink]
Can anybody explain why option B is not correct?
GMAT Club Reviews PM Intern
Joined: 10 Apr 2018
Posts: 532
Own Kudos [?]: 754 [0]
Given Kudos: 522
Location: India
Schools: ISB'22 (D)
GMAT 1: 680 Q48 V34
GPA: 3.3
Send PM
Editorial: Our city's public transportation agency is facing a budget [#permalink]
I selected option B because if the statement in it is not true, then that means there is/are other ways to resolve the deficit rather than to cut transportation services, causing loss in jobs, so in such a case there's no need for the employee union to accept cuts in retirement benefits.

Could anyone please tell me where's the gap in my reasoning?

Originally posted by siddharthkapoor on 28 Jun 2019, 01:19.
Last edited by siddharthkapoor on 10 Oct 2019, 01:44, edited 1 time in total.
Current Student
Joined: 15 Jun 2020
Posts: 319
Own Kudos [?]: 81 [0]
Given Kudos: 245
Location: United States
GPA: 3.3
Send PM
Editorial: Our city's public transportation agency is facing a budget [#permalink]
Hi experts, MartyTargetTestPrep GMATNinja AjiteshArun

The answer is definitely D, but I can't seem to understand exactly why A is incorrect. Is it because the choice flips employees' interest and the union's acceptance and negates both? Therefore, it talks about NOT accepting cuts in benefits, therefore straying away from the point of the argument - i.e., what the union should accept.

I originally ruled out this option because it states "the transportation employees' union SHOULD not accept cuts...if doing so...." Why should something that the union should or should not be assumed?
Intern
Intern
Joined: 11 May 2021
Posts: 9
Own Kudos [?]: 1 [0]
Given Kudos: 24
Send PM
Re: Editorial: Our city's public transportation agency is facing a budget [#permalink]
aggvipul wrote:
TheRzS wrote:
Could an expert please weigh in?

Q1.
A. The transportation employees' union should not accept cuts in retirement benefits if doing so would not be in the employees' best interest.
E. The transportation employees' union will not accept cuts in retirement benefits if doing so will not allow more transportation employees to keep their jobs.

In A and E, does "doing so" stand for ACCEPTING or NOT ACCEPTING?

Q2.
Could someone please try negating A and E? Negating them seems to hurt the argument.

Thanks and regards

TheRzS Q1 -"doing so" here is indicating "Accepting"
Q2- Negating the option A. rather warps the information or the intent of what is being said, try this
A1. The transportation employees' union should accept cuts in retirement benefits if doing so would not be in the employees' best interest- it rather now intents to say that employees should accept loss
A2. The transportation employees' union should not accept cuts in retirement benefits if doing so would be in the employees' best interest- same here, it intents to say that employees should accept loss
Therefore, it does not hurt the argument rather warps the gist of question stem
would be happy to help in case anything is still unclear


GMATNinja
generis
VeritasKarishma

Dear Experts,

Please suggest what negated statements would option A and E have,
since there are two verbs -('should not accept' and 'would not be in favour')

do we need to negate both or any one of them, and on what grounds should we negate the verb out of the two

thanks
VP
VP
Joined: 15 Dec 2016
Posts: 1374
Own Kudos [?]: 207 [0]
Given Kudos: 189
Send PM
Editorial: Our city's public transportation agency is facing a budget [#permalink]
Hi VeritasKarishma

Quote:
Option B) The only feasible way for the agency to resolve the budget shortfall would involve cutting transportation service and eliminating jobs.


When one negates B, negated B becomes

The only feasible way for the agency to resolve the budget shortfalf MAY OR MAY NOT involve cutting transportation service and eliminating jobs

Originally posted by jabhatta2 on 18 Nov 2021, 16:47.
Last edited by jabhatta2 on 18 Nov 2021, 17:34, edited 8 times in total.
Senior Manager
Senior Manager
Joined: 24 Dec 2022
Posts: 293
Own Kudos [?]: 149 [0]
Given Kudos: 222
Send PM
Re: Editorial: Our city's public transportation agency is facing a budget [#permalink]
dcummins wrote:
The answer choices makes it seem as if the argument is what is best for the agency not for the employees.

The official explanation reasoning states: "If cutting the employees' retirement benefits would not be sufficient to resolve the budget shortfall, then it may well not be in employees' best interest".

Either way, cutting benefits is never in an employees best interest, but now I see that if benefits are cut and the shortfall still isn't resolved then the Agency may still very well cut jobs, so this is why answer (C) is correct!

Where does it say this? ie, the Official explanation reasoning that you state.
Tutor
Joined: 16 Oct 2010
Posts: 14831
Own Kudos [?]: 64941 [0]
Given Kudos: 427
Location: Pune, India
Send PM
Re: Editorial: Our city's public transportation agency is facing a budget [#permalink]
Expert Reply
0Lucky0 wrote:
dcummins wrote:
The answer choices makes it seem as if the argument is what is best for the agency not for the employees.

The official explanation reasoning states: "If cutting the employees' retirement benefits would not be sufficient to resolve the budget shortfall, then it may well not be in employees' best interest".

Either way, cutting benefits is never in an employees best interest, but now I see that if benefits are cut and the shortfall still isn't resolved then the Agency may still very well cut jobs, so this is why answer (C) is correct!

Where does it say this? ie, the Official explanation reasoning that you state.


This reasoning is deduced.

We are given that there is a budget shortfall i.e. money is short.
Retirement benefits are very generous i.e. a lot of money is given to retirees.
If more money is not generated/saved, services will be cut and employees may start losing their jobs.

Hence, the conclusion is that reducing retirement benefits to save money (so that services are not cut and employees do not lose jobs) is in the best interests of employees.

The assumption of course is that reducing retirement benefits will lead to saving enough money so that services are not cut and employees do not lose jobs.
Otherwise, their retirement benefits will be cut and still jobs will be lost. So it will become a lose-lose for the employees.
VP
VP
Joined: 15 Dec 2016
Posts: 1374
Own Kudos [?]: 207 [0]
Given Kudos: 189
Send PM
Editorial: Our city's public transportation agency is facing a budget [#permalink]
Hi GMATNinja - how do you re-phrase (A) and (E) ?

The double negatives (marked in red) coupled with the "IF STATEMENT" [underline] is very hard to wrap your head around

Quote:
(A) The transportation employees' union should not accept cuts in retirement benefits if doing so would not be in the employees' best interest.
VP
VP
Joined: 15 Dec 2016
Posts: 1374
Own Kudos [?]: 207 [0]
Given Kudos: 189
Send PM
Editorial: Our city's public transportation agency is facing a budget [#permalink]
Public Transportation Budget

Step 1: Identify the Question

The word assumption in the question stem indicates that this is a Find the Assumption question.

Step 2: Deconstruct the Argument

Budget shortfall

ERB generous, fastest growing cost

Don’t fix budget → Cut service, job losses

© Accept cuts to ERB

Note: ERB is used as an abbreviation for employee retirement benefits.


Step 3: Pause and State the Goal


On Find the Assumption questions, the correct answer should be necessary to draw the conclusion. Remind yourself of the key logic of the argument: accepting cuts to retirement benefits is in the best interest of employees (because doing so will prevent job losses).


Step 4: Work from Wrong to Right

(A) The argument is not concerned with the motivations of the union or what the union should do in general. Although the conclusion states that the cuts would be in the employees’ best interest in this case, whether the union would accept the cuts for this reason or for some other reason (for example, the negotiation is easier) is not relevant.

(B) This answer weakens the conclusion. The conclusion is based on a belief that if the union accepts the benefit cuts the budget shortfall can be resolved without cutting service and jobs.

(C) If anything, this answer weakens the argument. If employees prefer generous retirement benefits, then it is less likely that accepting cuts to these benefits will be in their best interest.

(D) CORRECT. This answer establishes that the potential savings from cutting retirement benefits could actually resolve the budget shortfall (and subsequently prevent service cuts and job losses). While the argument states that employee retirement benefits are the fastest growing part of the budget, it does not discuss the portion of the budget retirement benefits make up. Consider the Negation Test for further proof; the negation of a correct answer will ruin the argument. The negation of this answer is “Cutting retirement benefits might not help resolve the budget shortfall.” If the budget shortfall is not resolved, then services get cut and jobs are lost: the plan will fail.

(E) The specific motivations of the union are not relevant. The argument depends on the union choosing to accept the cuts to retirement benefits, but the union could do so for a variety of reasons: to save jobs, to get raises for some employees, or because they are bad negotiators.

Originally posted by jabhatta2 on 05 Jan 2023, 08:18.
Last edited by jabhatta2 on 05 Jan 2023, 09:48, edited 1 time in total.
Senior Manager
Senior Manager
Joined: 24 Dec 2022
Posts: 293
Own Kudos [?]: 149 [0]
Given Kudos: 222
Send PM
Re: Editorial: Our city's public transportation agency is facing a budget [#permalink]
jabhatta2 wrote:
Hi GMATNinja - how do you re-phrase (A) and (E) ?

The double negatives (marked in red) coupled with the "IF STATEMENT" [underline] is very hard to wrap your head around

Quote:
(A) The transportation employees' union should not accept cuts in retirement benefits if doing so would not be in the employees' best interest.

Like this:
The transportation employees' union should accept cuts in retirement benefits "only" if doing so would be in the employees' best interest.
VP
VP
Joined: 15 Dec 2016
Posts: 1374
Own Kudos [?]: 207 [0]
Given Kudos: 189
Send PM
Editorial: Our city's public transportation agency is facing a budget [#permalink]
0Lucky0 wrote:
jabhatta2 wrote:
Hi GMATNinja - how do you re-phrase (A) and (E) ?

The double negatives (marked in red) coupled with the "IF STATEMENT" [underline] is very hard to wrap your head around

Quote:
(A) The transportation employees' union should not accept cuts in retirement benefits if doing so would not be in the employees' best interest.

Like this:
The transportation employees' union should accept cuts in retirement benefits "only" if doing so would be in the employees' best interest.


0Lucky0 - is the blue statement you wrote above, a re-phrase of (A) or is the blue statement a negation of A ?
VP
VP
Joined: 15 Dec 2016
Posts: 1374
Own Kudos [?]: 207 [0]
Given Kudos: 189
Send PM
Editorial: Our city's public transportation agency is facing a budget [#permalink]
KarishmaB wrote:
Hence the argument is not assuming that "cutting transportation & eliminating jobs" is the only way to resolve budget shortfall. It is suggesting that cutting retirement benefits is a way to resolve budget shortfall.

Also, when you negate (B), you get:

(B) The only feasible way for the agency to resolve the budget shortfall would involve cutting transportation service and eliminating jobs.
Negated (B) There are other feasible ways to resolve budget shortfall too.
.


Thank you so much KarishmaB. Just following up on negated B

Negated (B) There are other feasible ways to resolve budget shortfall too.

Just to crystalize my understanding of Negated B

Does the negation EXCLUDE "cutting transportation service and eliminating jobs" as a method to resolve the budget shortfall ?

I don't think Negated B excludes "cutting transportation service and eliminating jobs" as a method to resolve the budget shortfall, correct ?

Originally posted by jabhatta2 on 05 Jan 2023, 08:34.
Last edited by jabhatta2 on 05 Jan 2023, 09:43, edited 1 time in total.
Senior Manager
Senior Manager
Joined: 24 Dec 2022
Posts: 293
Own Kudos [?]: 149 [0]
Given Kudos: 222
Send PM
Re: Editorial: Our city's public transportation agency is facing a budget [#permalink]
jabhatta2 wrote:
Like this:
The transportation employees' union should accept cuts in retirement benefits "only" if doing so would be in the employees' best interest.

0Lucky0 - is the blue statement you wrote above, a re-phrase of (A) or is the blue statement a negation of A ?


re-phrase. Neat trick. Isn't it? :angel:
VP
VP
Joined: 15 Dec 2016
Posts: 1374
Own Kudos [?]: 207 [0]
Given Kudos: 189
Send PM
Editorial: Our city's public transportation agency is facing a budget [#permalink]
0Lucky0 wrote:
jabhatta2 wrote:
Like this:
The transportation employees' union should accept cuts in retirement benefits "only" if doing so would be in the employees' best interest.

0Lucky0 - is the blue statement you wrote above, a re-phrase of (A) or is the blue statement a negation of A ?


re-phrase. Neat trick. Isn't it? :angel:



Yes :angel: 0Lucky0 -

Could you go over the theory as to how you know about this ? what is the theory behind this ?

What do you think of my re-phrases in these simple analogies ?

Quote:
Analogy 1 : you should not drive a car if you havent passed your car exam
Re-phrase : you should drive ONLY IF YOU HAVE passed


Quote:
Analogy 2 : You should not sleep if you haven’t eaten
Re-phrase : You should sleep ONLY if you have eaten
Senior Manager
Senior Manager
Joined: 24 Dec 2022
Posts: 293
Own Kudos [?]: 149 [0]
Given Kudos: 222
Send PM
Editorial: Our city's public transportation agency is facing a budget [#permalink]
jabhatta2 wrote:
0Lucky0 wrote:
jabhatta2 wrote:
Like this:
The transportation employees' union should accept cuts in retirement benefits "only" if doing so would be in the employees' best interest.

0Lucky0 - is the blue statement you wrote above, a re-phrase of (A) or is the blue statement a negation of A ?


re-phrase. Neat trick. Isn't it? :angel:



Yes :angel:

Could you go over the theory as to how you know about this ? what is the theory behind this ?

What do you think of my re-phrases in these simple analogies ?

Quote:
Analogy 1 : you should not drive a car if you havent passed your car exam
Re-phrase : you should drive ONLY IF YOU HAVE passed


Quote:
Analogy 2 : You should not sleep if you haven’t eaten
Re-phrase : You should sleep ONLY if you have eaten

I am not sure if what you have posted is allowed here. I mean, it's off-topic. So would most likely look like spam to others.
VP
VP
Joined: 15 Dec 2016
Posts: 1374
Own Kudos [?]: 207 [0]
Given Kudos: 189
Send PM
Editorial: Our city's public transportation agency is facing a budget [#permalink]
Hi KarishmaB - is my negation accurate for (a) and (e) ?

theory :
If A, then B

Negation is – If A, then may or may not be B


(Option A)
If cuts in retirement benefits are not in the employees' best interest, union should not accept cuts in retirement benefits

(Negation A)
If cuts in retirement benefits are not in the employees' best interest, union may or may not accept cuts in retirement benefits

(Option E)
If cuts in retirement benefits will not allow more transportation employees to keep their jobs, the union will not accept cuts in retirement benefits.

(Negation E)
If cuts in retirement benefits will not allow more transportation employees to keep their jobs, the union may or may not accept cuts in retirement benefits.
GMAT Club Bot
Editorial: Our city's public transportation agency is facing a budget [#permalink]
   1   2   3   4   
Moderators:
GMAT Club Verbal Expert
6923 posts
GMAT Club Verbal Expert
238 posts
CR Forum Moderator
832 posts

Powered by phpBB © phpBB Group | Emoji artwork provided by EmojiOne